Difference between revisions of "2022 AMC 12A Problems/Problem 22"

 
(9 intermediate revisions by 3 users not shown)
Line 2: Line 2:
  
 
Let <math>c</math> be a real number, and let <math>z_1</math> and <math>z_2</math> be the two complex numbers satisfying the equation
 
Let <math>c</math> be a real number, and let <math>z_1</math> and <math>z_2</math> be the two complex numbers satisfying the equation
<math>z^2 - cz + 10 = 0</math>. Points <math>z_1</math>, <math>z_2</math>, <math>\frac{1}{z_1}</math>, and <math>\frac{1}{z_2}</math> are the vertices of (convex) quadrilateral <math>Q</math> in the complex plane. When the area of <math>Q</math> obtains its maximum possible value, <math>c</math> is closest to which of the following?
+
<math>z^2 - cz + 10 = 0</math>. Points <math>z_1</math>, <math>z_2</math>, <math>\frac{1}{z_1}</math>, and <math>\frac{1}{z_2}</math> are the vertices of (convex) quadrilateral <math>\mathcal{Q}</math> in the complex plane. When the area of <math>\mathcal{Q}</math> obtains its maximum possible value, <math>c</math> is closest to which of the following?
  
 
<math>\textbf{(A) }4.5 \qquad\textbf{(B) }5 \qquad\textbf{(C) }5.5 \qquad\textbf{(D) }6\qquad\textbf{(E) }6.5</math>
 
<math>\textbf{(A) }4.5 \qquad\textbf{(B) }5 \qquad\textbf{(C) }5.5 \qquad\textbf{(D) }6\qquad\textbf{(E) }6.5</math>
Line 26: Line 26:
 
c & = z_1 + z_2 \\
 
c & = z_1 + z_2 \\
 
& = z_1 + \bar z_1 \\
 
& = z_1 + \bar z_1 \\
& = 2 {\rm Re} \ z_1 ,
+
& = 2 {\rm Re}(z_1),
 
\end{align*}
 
\end{align*}
 
</cmath>
 
</cmath>
 
where the first equality follows from Vieta's formula.
 
where the first equality follows from Vieta's formula.
  
Thus, <math>{\rm Re} \ z_1 = \frac{c}{2}</math>.
+
Thus, <math>{\rm Re}(z_1) = \frac{c}{2}</math>.
  
 
We have
 
We have
 
<cmath>
 
<cmath>
 
\begin{align*}
 
\begin{align*}
\frac{1}{z_1} & = \frac{1}{10} \frac{10}{z_1} \\
+
\frac{1}{z_1} & = \frac{1}{10}\cdot\frac{10}{z_1} \\
& = \frac{1}{10} \frac{z_1 z_2}{z_1} \\
+
& = \frac{1}{10}\cdot\frac{z_1 z_2}{z_1} \\
 
& = \frac{z_2}{10} \\
 
& = \frac{z_2}{10} \\
& = \frac{\bar z_1}{10} .
+
& = \frac{\bar z_1}{10}.
 
\end{align*}
 
\end{align*}
 
</cmath>
 
</cmath>
Line 47: Line 47:
 
<cmath>
 
<cmath>
 
\begin{align*}
 
\begin{align*}
\frac{1}{z_2} & = \frac{1}{10} \frac{10}{z_2} \\
+
\frac{1}{z_2} & = \frac{1}{10}\cdot\frac{10}{z_2} \\
& = \frac{1}{10} \frac{z_1 z_2}{z_2} \\
+
& = \frac{1}{10}\cdot\frac{z_1 z_2}{z_2} \\
& = \frac{z_1}{10} .
+
& = \frac{z_1}{10}.
 
\end{align*}
 
\end{align*}
 
</cmath>
 
</cmath>
Line 58: Line 58:
 
\begin{align*}
 
\begin{align*}
 
{\rm Area} \ Q
 
{\rm Area} \ Q
& = \frac{1}{2} \left| {\rm Re} \ z_1 \right|
+
& = \frac{1}{2} \left| {\rm Re}(z_1) \right|
\cdot 2 \left| {\rm Im} \ z_1 \right|
+
\cdot 2 \left| {\rm Im}(z_1) \right|
 
\cdot \left( 1 - \frac{1}{10^2} \right) \\
 
\cdot \left( 1 - \frac{1}{10^2} \right) \\
 
& = \frac{1}{2} |c| \sqrt{10 - \frac{c^2}{4}} \left( 1 - \frac{1}{10^2} \right) \\
 
& = \frac{1}{2} |c| \sqrt{10 - \frac{c^2}{4}} \left( 1 - \frac{1}{10^2} \right) \\
Line 73: Line 73:
 
~Steven Chen (Professor Chen Education Palace, www.professorchenedu.com)
 
~Steven Chen (Professor Chen Education Palace, www.professorchenedu.com)
  
==Solution 2==
+
==Solution 2 (Incomplete)==
 
Because <math>z^2 - cz + 10 = 0</math>, notice that <math>|z_1||z_2|=|10|=10</math>. Furthermore, note that because <math>c</math> is real, <math>z_2=\bar z_1</math>. Thus, <math>\frac{1}{z_1}=\frac{\bar z_1}{z_1\cdot{\bar z_1}}=\frac{z_2}{|z_1|^2}=\frac{z_2}{100}</math>. Similarly, <math>\frac{1}{z_2}=\frac{z_1}{100}</math>. On the complex coordinate plane, let <math>z_1=A_2</math>, <math>z_2=B_2</math>,<math>\frac{1}{z_2}=A_1</math>, <math>\frac{1}{z_1}=B_1</math>. Notice how <math>OA_1B_1</math> is similar to <math>OA_2B_2</math>. Thus, the area of <math>A_1B_1B_2B_1</math> is  <math>(k)(OA_2B_2)</math> for some constant <math>k</math>, and <math>OA_2B_2 = </math>
 
Because <math>z^2 - cz + 10 = 0</math>, notice that <math>|z_1||z_2|=|10|=10</math>. Furthermore, note that because <math>c</math> is real, <math>z_2=\bar z_1</math>. Thus, <math>\frac{1}{z_1}=\frac{\bar z_1}{z_1\cdot{\bar z_1}}=\frac{z_2}{|z_1|^2}=\frac{z_2}{100}</math>. Similarly, <math>\frac{1}{z_2}=\frac{z_1}{100}</math>. On the complex coordinate plane, let <math>z_1=A_2</math>, <math>z_2=B_2</math>,<math>\frac{1}{z_2}=A_1</math>, <math>\frac{1}{z_1}=B_1</math>. Notice how <math>OA_1B_1</math> is similar to <math>OA_2B_2</math>. Thus, the area of <math>A_1B_1B_2B_1</math> is  <math>(k)(OA_2B_2)</math> for some constant <math>k</math>, and <math>OA_2B_2 = </math>
 
(In progress)
 
(In progress)
Line 92: Line 92:
 
With the restriction that <math>a^2+b^2=(a-b)^2+2ab=10</math>, <math>ab</math> is maximized when <math>a=b=\sqrt{5}</math>.  
 
With the restriction that <math>a^2+b^2=(a-b)^2+2ab=10</math>, <math>ab</math> is maximized when <math>a=b=\sqrt{5}</math>.  
  
Remember, <math>c</math> is the sum of the roots, so <math>c=z_1+z_2=2a=2\sqrt5=\sqrt{20}\approx\boxed{4.5}</math> ~quacker88
+
Remember, <math>c</math> is the sum of the roots, so <math>c=z_1+z_2=2a=2\sqrt5=\sqrt{20}\approx\boxed{4.5}</math>  
 +
 
 +
~quacker88
  
 
==Solution 4 (Fast)==
 
==Solution 4 (Fast)==
Line 99: Line 101:
 
Let <math>z_1=\sqrt{10}e^{i\theta}</math> where <math>0<\theta<\pi</math>, then <math>z_2=\sqrt{10}e^{-i\theta}</math>, <math>\frac{1}{z_1}=\frac{1}{\sqrt{10}}e^{-i\theta} </math>, <math>\frac{1}{z_2}= \frac{1}{\sqrt{10}}e^{i\theta}</math>.
 
Let <math>z_1=\sqrt{10}e^{i\theta}</math> where <math>0<\theta<\pi</math>, then <math>z_2=\sqrt{10}e^{-i\theta}</math>, <math>\frac{1}{z_1}=\frac{1}{\sqrt{10}}e^{-i\theta} </math>, <math>\frac{1}{z_2}= \frac{1}{\sqrt{10}}e^{i\theta}</math>.
  
On the basis of symmetry, the area <math>A</math> of <math>Q</math> is the difference between two isoceles triangles,so
+
On the basis of symmetry, the area <math>A</math> of <math>\mathcal{Q}</math> is the difference between two isoceles triangles,so
 +
 
 +
<math>2A=10\sin2\theta-\frac{1}{10}\sin2\theta\leq10-\frac{1}{10}</math>. The inequality holds when <math>2\theta=\frac{\pi}{2}</math>, or <math>\theta=\frac{\pi}{4}</math>.
 +
 
 +
Thus, <math>c= 2 {\rm Re} \ z_1 =2 \sqrt{10} \cos\frac{\pi}{4}=\sqrt{20} \approx \boxed{\textbf{(A) 4.5}}</math>.
 +
 
 +
~PluginL
 +
 
 +
==Solution 5 (Calculus Finish)==
 +
 
 +
Like in Solution 3, we find that <math>Q = \frac12\cdot\left(2b+\frac{b}{5}\right)\cdot\frac{9a}{10}= \frac{99}{100}ab</math>, thus, <math>Q</math> is maximized when <math>ab</math> is maximized. <math>ab = a \cdot \sqrt{10 - a^2} = \sqrt{10a^2 - a^4}</math>, let <math>f(a) =  \sqrt{10a^2 - a^4}</math>.
 +
 
 +
By the Chain Rule and the Power Rule, <math>f'(a) = \frac12 \cdot (10a^2 - a^4)^{-\frac12} \cdot (10(2a)-4a^3)) = \frac{20a-4a^3}{ 2\sqrt{10a^2 - a^4} } = \frac{10a-2a^3}{ \sqrt{10a^2 - a^4} }</math>
 +
 
 +
<math>f'(a) = 0</math>, <math>10a-2a^3 = 0</math>, <math>a \neq 0</math>, <math>a^2 = 5</math>, <math>a = \sqrt{5}</math>
 +
 
 +
<math>\because f'(a) = 0</math> when <math>a = \sqrt{5}</math>, <math>f'(a)</math> is positive when <math>a < \sqrt{5}</math>, and <math>f'(a)</math> is negative when <math>a > \sqrt{5}</math>
 +
 
 +
<math>\therefore f(a)</math> has a local maximum when <math>a = \sqrt{5}</math>.
 +
 
 +
Notice that <math>ab = \frac{ (a+b)^2 - (a^2+b^2) }{2} = \frac{c^2 - 10}{2}</math>, <math>\frac{c^2 - 10}{2} = 5</math>, <math>c = \sqrt{2 \cdot 5 + 10} = \sqrt{20} \approx \boxed{\text{(A) 4.5}}</math>
  
<math>2A=10sin2\theta-\frac{1}{10}sin2\theta\leq10-\frac{1}{10}</math>. The inequality holds when <math>2\theta=\frac{\pi}{2}</math>, or <math>\theta=\frac{\pi}{4}</math>.
+
~[https://artofproblemsolving.com/wiki/index.php/User:Isabelchen isabelchen]
  
Thus, <math>c= 2 {\rm Re} \ z_1 =2 \sqrt{10} cos\frac{\pi}{4}=\sqrt{20} \approx \boxed{\textbf{(A) 4.5}} </math> ~PluginL
+
==Video Solution by Math-X (Smart and Simple)==
 +
https://youtu.be/7yAh4MtJ8a8?si=1NLsu57rYrEPP1A2&t=6893 ~Math-X
  
 
==Video Solution by Punxsutawney Phil==
 
==Video Solution by Punxsutawney Phil==

Latest revision as of 11:20, 1 January 2024

Problem

Let $c$ be a real number, and let $z_1$ and $z_2$ be the two complex numbers satisfying the equation $z^2 - cz + 10 = 0$. Points $z_1$, $z_2$, $\frac{1}{z_1}$, and $\frac{1}{z_2}$ are the vertices of (convex) quadrilateral $\mathcal{Q}$ in the complex plane. When the area of $\mathcal{Q}$ obtains its maximum possible value, $c$ is closest to which of the following?

$\textbf{(A) }4.5 \qquad\textbf{(B) }5 \qquad\textbf{(C) }5.5 \qquad\textbf{(D) }6\qquad\textbf{(E) }6.5$

Solution 1

Because $c$ is real, $z_2 = \bar z_1$. We have \begin{align*} 10 & = z_1 z_2 \\ & = z_1 \bar z_1 \\ & = |z_1|^2 , \end{align*} where the first equality follows from Vieta's formula.

Thus, $|z_1| = \sqrt{10}$.

We have \begin{align*} c & = z_1 + z_2 \\ & = z_1 + \bar z_1 \\ & = 2 {\rm Re}(z_1), \end{align*} where the first equality follows from Vieta's formula.

Thus, ${\rm Re}(z_1) = \frac{c}{2}$.

We have \begin{align*} \frac{1}{z_1} & = \frac{1}{10}\cdot\frac{10}{z_1} \\ & = \frac{1}{10}\cdot\frac{z_1 z_2}{z_1} \\ & = \frac{z_2}{10} \\ & = \frac{\bar z_1}{10}. \end{align*} where the second equality follows from Vieta's formula.

We have \begin{align*} \frac{1}{z_2} & = \frac{1}{10}\cdot\frac{10}{z_2} \\ & = \frac{1}{10}\cdot\frac{z_1 z_2}{z_2} \\ & = \frac{z_1}{10}. \end{align*} where the second equality follows from Vieta's formula.

Therefore, \begin{align*} {\rm Area} \ Q & = \frac{1}{2} \left| {\rm Re}(z_1) \right| \cdot 2 \left| {\rm Im}(z_1) \right| \cdot \left( 1 - \frac{1}{10^2} \right) \\ & = \frac{1}{2} |c| \sqrt{10 - \frac{c^2}{4}} \left( 1 - \frac{1}{10^2} \right) \\ & = \frac{1 - \frac{1}{10^2}}{4} \sqrt{c^2 \left( 40 - c^2 \right)} \\ & \leq \frac{1 - \frac{1}{10^2}}{4} \cdot \frac{c^2 + \left( 40 - c^2 \right)}{2} \\ & = \frac{1 - \frac{1}{10^2}}{4} \cdot 20 , \end{align*} where the inequality follows from the AM-GM inequality, and it is augmented to an equality if and only if $c^2 = 40 - c^2$. Thus, $|c| = 2 \sqrt{5} \approx \boxed{\textbf{(A) 4.5}}$.

~Steven Chen (Professor Chen Education Palace, www.professorchenedu.com)

Solution 2 (Incomplete)

Because $z^2 - cz + 10 = 0$, notice that $|z_1||z_2|=|10|=10$. Furthermore, note that because $c$ is real, $z_2=\bar z_1$. Thus, $\frac{1}{z_1}=\frac{\bar z_1}{z_1\cdot{\bar z_1}}=\frac{z_2}{|z_1|^2}=\frac{z_2}{100}$. Similarly, $\frac{1}{z_2}=\frac{z_1}{100}$. On the complex coordinate plane, let $z_1=A_2$, $z_2=B_2$,$\frac{1}{z_2}=A_1$, $\frac{1}{z_1}=B_1$. Notice how $OA_1B_1$ is similar to $OA_2B_2$. Thus, the area of $A_1B_1B_2B_1$ is $(k)(OA_2B_2)$ for some constant $k$, and $OA_2B_2 =$ (In progress)

Solution 3 (Trapezoid)

Since $c$, which is the sum of roots $z_1$ and $z_2$, is real, $z_1=\overline{z_2}$.

Let $z_1=a+bi$. Then $z_2=a-bi$. Note that the product of the roots is $10$ by Vieta's, so $z_1z_2=(a+bi)(a-bi)=a^2+b^2=10$.

Thus, $\frac{1}{z_1}=\frac{1}{a-bi}\cdot\frac{a+bi}{a+bi}=\frac{a+bi}{a^2+b^2}=\frac{a+bi}{10}$. With the same process, $\frac{1}{z_2}=\frac{a-bi}{10}$.

So, our four points are $a+bi,\frac{a+bi}{10},a-bi,$ and $\frac{a-bi}{10}$. WLOG let $a+bi$ be in the first quadrant and graph these four points on the complex plane. Notice how quadrilateral Q is a trapezoid with the real axis as its axis of symmetry. It has a short base with endpoints $\frac{a+bi}{10}$ and $\frac{a-bi}{10}$, so its length is $\left|\frac{a+bi}{10} - \frac{a-bi}{10}\right| =\frac{b}{5}$. Likewise, its long base has endpoints $a+bi$ and $a-bi$, so its length is $|(a+bi)-(a-bi)|=2b$.

The height, which is the distance between the two lines, is the difference between the real values of the two bases $\implies h= a-\frac{a}{10}=\frac{9a}{10}$.

Plugging these into the area formula for a trapezoid, we are trying to maximize $\frac12\cdot\left(2b+\frac{b}{5}\right)\cdot\frac{9a}{10}=\frac{99ab}{100}$. Thus, the only thing we need to maximize is $ab$.

With the restriction that $a^2+b^2=(a-b)^2+2ab=10$, $ab$ is maximized when $a=b=\sqrt{5}$.

Remember, $c$ is the sum of the roots, so $c=z_1+z_2=2a=2\sqrt5=\sqrt{20}\approx\boxed{4.5}$

~quacker88

Solution 4 (Fast)

Like the solutions above we can know that $|z_1| = |z_2| = \sqrt{10}$ and $z_1=\overline{z_2}$.

Let $z_1=\sqrt{10}e^{i\theta}$ where $0<\theta<\pi$, then $z_2=\sqrt{10}e^{-i\theta}$, $\frac{1}{z_1}=\frac{1}{\sqrt{10}}e^{-i\theta}$, $\frac{1}{z_2}= \frac{1}{\sqrt{10}}e^{i\theta}$.

On the basis of symmetry, the area $A$ of $\mathcal{Q}$ is the difference between two isoceles triangles,so

$2A=10\sin2\theta-\frac{1}{10}\sin2\theta\leq10-\frac{1}{10}$. The inequality holds when $2\theta=\frac{\pi}{2}$, or $\theta=\frac{\pi}{4}$.

Thus, $c= 2 {\rm Re} \ z_1 =2 \sqrt{10} \cos\frac{\pi}{4}=\sqrt{20} \approx \boxed{\textbf{(A) 4.5}}$.

~PluginL

Solution 5 (Calculus Finish)

Like in Solution 3, we find that $Q = \frac12\cdot\left(2b+\frac{b}{5}\right)\cdot\frac{9a}{10}= \frac{99}{100}ab$, thus, $Q$ is maximized when $ab$ is maximized. $ab = a \cdot \sqrt{10 - a^2} = \sqrt{10a^2 - a^4}$, let $f(a) =  \sqrt{10a^2 - a^4}$.

By the Chain Rule and the Power Rule, $f'(a) = \frac12 \cdot (10a^2 - a^4)^{-\frac12} \cdot (10(2a)-4a^3)) = \frac{20a-4a^3}{ 2\sqrt{10a^2 - a^4} } = \frac{10a-2a^3}{ \sqrt{10a^2 - a^4} }$

$f'(a) = 0$, $10a-2a^3 = 0$, $a \neq 0$, $a^2 = 5$, $a = \sqrt{5}$

$\because f'(a) = 0$ when $a = \sqrt{5}$, $f'(a)$ is positive when $a < \sqrt{5}$, and $f'(a)$ is negative when $a > \sqrt{5}$

$\therefore f(a)$ has a local maximum when $a = \sqrt{5}$.

Notice that $ab = \frac{ (a+b)^2 - (a^2+b^2) }{2} = \frac{c^2 - 10}{2}$, $\frac{c^2 - 10}{2} = 5$, $c = \sqrt{2 \cdot 5 + 10} = \sqrt{20} \approx \boxed{\text{(A) 4.5}}$

~isabelchen

Video Solution by Math-X (Smart and Simple)

https://youtu.be/7yAh4MtJ8a8?si=1NLsu57rYrEPP1A2&t=6893 ~Math-X

Video Solution by Punxsutawney Phil

https://youtube.com/watch?v=bbMcdvlPcyA

Video Solution by Steven Chen

https://youtu.be/pcB2sg7Ag58

~Steven Chen (Professor Chen Education Palace, www.professorchenedu.com)

See Also

2022 AMC 12A (ProblemsAnswer KeyResources)
Preceded by
Problem 21
Followed by
Problem 23
1 2 3 4 5 6 7 8 9 10 11 12 13 14 15 16 17 18 19 20 21 22 23 24 25
All AMC 12 Problems and Solutions

The problems on this page are copyrighted by the Mathematical Association of America's American Mathematics Competitions. AMC logo.png